PT14.S2.Q21 - In response to high mortality in area hospitals

wkim2015wkim2015 Member
edited June 2018 in Logical Reasoning 86 karma

Does anybody have any idea why the answer choice is (a) for this question? I couldn't find an answer so I ended up eliminating all of the answer choices because none of them seemed to make sense. Answer choice (a) seems to strengthen the argument, if anything. Someone please help! Thank you!

Comments

  • keets993keets993 Alum Member 🍌
    6045 karma

    Hey, so according to the question bank, PT21.S2.Q21 is parallel flaw is that the you're asking about? Since you mentioned that a seems to strengthen the argument

  • wkim2015wkim2015 Member
    86 karma

    sorry about that. I fixed it!

  • Nabintou-1Nabintou-1 Alum Member
    edited June 2018 410 karma

    this question's really confusing. I chose "A" because it was the only one that resisted the idea that the risks were unnecessary (conc.).

  • paulmv.benthempaulmv.benthem Alum Member
    1032 karma

    Here's the thought processes I had when doing this question. The conclusion of the argument is that doctors were taking unnecessary risks by performing elective surgeries (which could could be temporarily postponed). The author supports this by showing that the mortality rate dropped in the hospital when elective surgeries were suspended. "A' seems to challenge the argument by noting that NOT doing the surgeries can lead to more problematic issues down the line. In other words, it introduces an argument supporting elective surgeries, despite the risks, because not doing so could be riskier. Ultimately, this challenges the notion that the risks were unnecessary, which is what the author suggests. It doesn't refute it, but it does shed doubt on the inference that the author is making based on the hospital stats.

    Hope that helps!

Sign In or Register to comment.